LSAT and Law School Admissions Forum

Get expert LSAT preparation and law school admissions advice from PowerScore Test Preparation.

 MikeJones
  • Posts: 31
  • Joined: Oct 02, 2017
|
#41249
Can someone help explain how the conditional logic in the stimulus supports answer D?

In the first sentence, I see:
A :arrow: B

In the second sentence, I see:
C :arrow: D :arrow: /B :arrow: /A

The correct AC, (D), states that /B :arrow: C

I'm just not sure what I'm missing here. Thanks.
 Eric Ockert
PowerScore Staff
  • PowerScore Staff
  • Posts: 164
  • Joined: Sep 28, 2011
|
#41462
Hi Mike!

I think your analysis on answer choice (D) is accurate. But answer choice (D) is not the correct answer here. Answer choice (C) is actually the correct answer. That may be what you are missing.

Hope that helps!
 MikeJones
  • Posts: 31
  • Joined: Oct 02, 2017
|
#41603
Eric Ockert wrote:Hi Mike!

I think your analysis on answer choice (D) is accurate. But answer choice (D) is not the correct answer here. Answer choice (C) is actually the correct answer. That may be what you are missing.

Hope that helps!
Thank you, Eric. You can delete the thread if you want.
 dandelionsroar
  • Posts: 27
  • Joined: Oct 18, 2018
|
#72099
Hi,

I was between C and E, picked he because it sounded better but was annoyed that I could not definitely rule out E. After looking at it again I think its incorrect because it is a mistaken negation of the stim but can you verify if my logic is correct?

Stim:
Prop Channeled->Due Weight

Govt control-> poltx-> ~prop channeled

the unstated assumption being poltx->~Due Weight

Answer E says ~govt. control->~poltx which is a mistaken negation(or mistaken reversal depending on how you see it) of the above and therefore incorrect? Did I diagram everything correctly?

thanks!
 Rachael Wilkenfeld
PowerScore Staff
  • PowerScore Staff
  • Posts: 1358
  • Joined: Dec 15, 2011
|
#72159
Hi dandelionsroar,

It looks like you have some of the conditional relationships a bit confused.

The first conditional is

Give due weight to scientific merits :arrow: scientific research properly channeled

This is because the due weight is modified by a sufficient condition indicator (whenever).

Your next conditional was spot on:

Government agencies control funding :arrow: political considerations major role :arrow: scientific research properly channeled

You were also absolutely right about answer choice (E)---it's a Mistaken Negation of the second conditional.
Government agencies control funding :arrow: political considerations major role

Hope that helps!
Rachael

Get the most out of your LSAT Prep Plus subscription.

Analyze and track your performance with our Testing and Analytics Package.